1 Introduction

The subject of fractional calculus has gained significant interest and been a valuable tool for both science and engineering (see [13]). In recent years, the fractional boundary value problems (FBVPs for short) have been considered by many authors (see [410] and the references therein). For example, Bai studied a FBVP at non-resonance with \(1<\alpha\leq2\) (see [10]). FBVPs at resonance were studied by Kosmatov (see [11]) and Jiang (see [12]). But the positive solutions for FBVPs at resonance were studied very few. In [13], Yang and Wang considered the positive solutions of the following FBVP:

$$\begin{aligned} \left \{ \textstyle\begin{array}{@{}l} -D_{0^{+}}^{\alpha}x(t)=f(t,x(t)), \quad t\in[0,1], \\ x(0)=0, \qquad x'(0)=x'(1). \end{array}\displaystyle \right . \end{aligned}$$

In [14], Chen and Tang studied the positive solution of FBVP as follows:

$$\begin{aligned} \left \{ \textstyle\begin{array}{@{}l} D_{0^{+}}^{\alpha}x(t)=f(t,x(t)),\quad t\in[0,+\infty), \\ x(0)=x'(0)=x''(0)=0, \qquad D_{0^{+}}^{\alpha-1}x(0)=\lim_{t\rightarrow +\infty}D_{0^{+}}^{\alpha-1}x(t). \end{array}\displaystyle \right . \end{aligned}$$

However, to the best of our knowledge, the fractional differential equations with m-point boundary conditions at resonance have not been considered. Motivated by the papers above, we consider the existence of positive solutions for a m-point FBVP of the form

$$\begin{aligned} \left \{ \textstyle\begin{array}{@{}l} -D_{0^{+}}^{\alpha}x(t)=f(t,x(t)), \quad t\in[0,1], \\ x'(0)=0, \qquad x(1)=\sum_{i=1}^{m-2}\beta_{i} x(\eta_{i}), \end{array}\displaystyle \right . \end{aligned}$$
(1.1)

where \(D_{0^{+}}^{\alpha}\) denotes the standard Caputo fractional differential operator of order α, \(1< \alpha\leq2\), \(\beta_{i} \in\mathbb{R^{+}}\), \(\sum_{i=1}^{m-2}\beta _{i}=1\), \(0<\eta_{1}<\eta_{2}<\cdots<\eta_{m-2}<1\), and \(f:[0,1]\times \mathbb{R} \rightarrow\mathbb{R} \) is continuous. Obviously, FBVP (1.1) happens to be at resonance under the condition \(\sum_{i=1}^{m-2}\beta_{i}=1\).

The rest of this paper is organized as follows. Section 2 contains some necessary notations, definitions and lemmas. In Section 3, we establish a theorem on the existence of positive solutions for FBVP (1.1) under some restrictions of f, basing on the coincidence degree theory due to [15]. Finally, in Section 4, an example is given to illustrate the main result.

2 Preliminaries

For convenience of the reader, we present some definitions, notations, and preliminary statements, which can be found in [2, 16, 17].

Let X and Y be real Banach spaces, \(L : \operatorname{dom} L\subset X\rightarrow Y\) be a Fredholm operator with index zero, where the index of a Fredholm operator L is defined by

$$\begin{aligned} \operatorname{Index}L : =\operatorname{dim} \operatorname{Ker}L- \operatorname{dim} \operatorname{Coker}L. \end{aligned}$$

Suppose \(P: X\rightarrow X\), \(Q: Y\rightarrow Y\) be continuous linear projectors such that

$$\begin{aligned}& \operatorname{Im} P=\operatorname{Ker} L, \qquad \operatorname{Ker} Q= \operatorname{Im} L, \\& X=\operatorname{Ker} L\oplus\operatorname{Ker} P, \qquad Y=\operatorname{Im} L \oplus\operatorname{Im} Q. \end{aligned}$$

Thus, we see that

$$\begin{aligned} L|_{\operatorname{dom} L\cap\operatorname{Ker} P}: \operatorname{dom} L\cap\operatorname{Ker} P\rightarrow \operatorname{Im} L \end{aligned}$$

is invertible. We denote the inverse by \(K_{P}\). Moreover, by virtue of \(\operatorname{dim} \operatorname{Im}Q=\operatorname{codim} \operatorname{Im}L\), there exists an isomorphism \(J: \operatorname{Im}Q\rightarrow\operatorname{Ker}L\). Then we know that the operator equation \(Lx=Nx\) is equivalent to

$$x=(P+JQN)x+K_{P}(I-Q)Nx, $$

where \(N: X\rightarrow Y\) be a nonlinear operator.

If Ω is an open bounded subset of X such that \(\operatorname{dom} L\cap\overline{\Omega} \neq\emptyset\), then the map \(N:X\rightarrow Y\) will be called L-compact on \(\overline{\Omega}\) if \(QN:\overline {\Omega}\rightarrow Y\) is bounded and \(K_{P}(I-Q)N:\overline{\Omega }\rightarrow X\) is compact.

Let C be a cone in X. Then C induces a partial order in X by

$$x\leq y \quad\mbox{if and only if} \quad y-x\in C. $$

Lemma 2.1

(see [15])

Let C be a cone in X. Then for every \(u\in C\setminus\{0\}\) there exists a positive number \(\sigma(u)\) such that

$$\|x+u\|\geq\sigma(u)\|x\| $$

for all \(x\in C\).

Let \(\gamma: X\rightarrow C\) be a retraction, that is, a continuous mapping such that \(\gamma(x)=x\) for all \(x\in C\). Set

$$\Psi:=P+JQN+K_{P}(I-Q)N $$

and

$$\Psi_{\gamma}:= \Psi\circ\gamma. $$

Lemma 2.2

(see [15])

Let C be a cone in X and \(\Omega_{1}\), \(\Omega_{2}\) be open bounded subsets of X with \(\overline{\Omega}_{1} \subset\Omega_{2}\) and \(C\cap (\overline{\Omega}_{2}\setminus\Omega_{1})\neq\emptyset\). Assume that the following conditions are satisfied:

  1. (1)

    \(L : \operatorname{dom} L\subset X\rightarrow Y\) be a Fredholm operator of index zero and \(N:X\rightarrow Y\) be L-compact on every bounded subset of X,

  2. (2)

    \(Lx\neq\lambda Nx\) for every \((x,\lambda)\in[C\cap\partial \Omega_{2} \cap\operatorname{dom} L]\times(0,1)\),

  3. (3)

    γ maps subsets of \(\overline{\Omega}_{2}\) into bounded subsets of C,

  4. (4)

    \(\operatorname{deg}([I-(P+JQN)\gamma]|_{\operatorname{Ker} L }, \operatorname{Ker} L \cap \Omega_{2} , 0)\neq0\),

  5. (5)

    there exists \(u_{0}\in C\setminus\{0\}\) such that \(\|x\| \leq \sigma(u_{0})\|\Psi x\|\) for \(x \in C(u_{0})\cap\partial\Omega_{1}\), where \(C(u_{0})=\{x\in C: \mu u_{0} \leq x \textit{ for some } \mu>0\}\) and \(\sigma(u_{0})\) is such that \(\|x+u_{0}\|\geq\sigma(u_{0})\|x\|\) for every \(x\in C\),

  6. (6)

    \((P+JQN)\gamma(\partial\Omega_{2}) \subset C\),

  7. (7)

    \(\Psi_{\gamma}(\overline{\Omega}_{2}\setminus\Omega_{1})\subset C\).

Then the equation \(Lx=Nx\) has at least one solution in \(C\cap(\overline {\Omega}_{2}\setminus\Omega_{1})\).

Definition 2.3

(see [17])

The Riemann-Liouville fractional integral operator of order \(\alpha>0\) of a function x is given by

$$\begin{aligned} I_{0^{+}}^{\alpha}x(t)=\frac{1}{\Gamma(\alpha)}\int_{0}^{t} (t-s)^{\alpha-1}x(s)\,ds, \end{aligned}$$

provided that the right side integral is pointwise defined on \((0,+\infty)\).

Definition 2.4

(see [17])

The Caputo fractional derivative of order \(\alpha>0\) of a continuous function x is given by

$$\begin{aligned} D_{0^{+}}^{\alpha}x(t)=\frac{1}{\Gamma(n-\alpha)}\int_{0}^{t}(t-s)^{n-\alpha -1} x^{(n)}(s)\,ds, \end{aligned}$$

where n is the smallest integer greater than or equal to α, provided that the right side integral is pointwise defined on \((0,+\infty)\).

Lemma 2.5

(see [18])

For \(\alpha>0\), the general solution of the Caputo fractional differential equation

$$\begin{aligned} D_{0^{+}}^{\alpha}x(t)=0 \end{aligned}$$

is

$$\begin{aligned} x(t)=c_{0}+c_{1}t+c_{2}t^{2}+ \cdots+c_{n-1}t^{n-1}, \end{aligned}$$

where \(c_{i}\in{\mathbb{R}}\), \(i=0,1,\ldots,n-1\), here n is the smallest integer greater than or equal to α.

Lemma 2.6

(see [18])

Suppose that \(D_{0^{+}}^{\alpha}x\in C[0,1]\), \(\alpha>0\). Then

$$\begin{aligned} I_{0^{+}}^{\alpha} D_{0^{+}}^{\alpha}x(t)=x(t)+c_{0}+c_{1}t+c_{2}t^{2}+ \cdots +c_{n-1}t^{n-1}, \end{aligned}$$

where \(c_{i}\in{\mathbb{R}}\), \(i=0,1,\ldots,n-1\), here n is the smallest integer greater than or equal to α.

In this paper, we take \(X=Y=C[0,1]\) with the norm \(\|x\|_{\infty}=\max_{t\in[0,1]} |x(t)|\).

Define the operator \(L:\operatorname{dom}L\subset X\rightarrow Y\) by

$$\begin{aligned} Lx=-D_{0^{+}}^{\alpha}x, \end{aligned}$$
(2.1)

where

$$\begin{aligned} \operatorname{dom} L=\Biggl\{ x\in X: D_{0^{+}}^{\alpha}x\in Y, x'(0)=0, x(1)=\sum_{i=1}^{m-2} \beta_{i} x(\eta_{i})\Biggr\} . \end{aligned}$$

Let \(N:X\rightarrow Y\) be the Nemytskii operator

$$\begin{aligned} Nx(t)=f\bigl(t,x(t)\bigr), \quad \forall t\in[0,1]. \end{aligned}$$

Then FBVP (1.1) is equivalent to the operator equation

$$\begin{aligned} Lx=Nx,\quad x\in\operatorname{dom}L. \end{aligned}$$

3 Main result

In this section, a theorem on the existence of positive solutions for FBVP (1.1) will be given.

For simplicity of notation, we set

$$ l_{i}(s)= \textstyle\begin{cases} (1-s)^{\alpha-1}-(\eta_{i}-s)^{\alpha-1}, & 0\leq s\leq\eta _{i} \leq1,\\ (1-s)^{\alpha-1}, & 0\leq\eta_{i} \leq s\leq1, \end{cases} $$

and

$$ G(t,s)= \textstyle\begin{cases} \frac{1}{\Gamma(\alpha+1)}(1-s)^{\alpha}\\ \quad{}+\frac{\alpha(\Gamma(\alpha+2)-1+(\alpha+1)t^{\alpha})}{\Gamma (\alpha+2)\sum_{i=1}^{m-2}\beta_{i}(1-\eta_{i}^{\alpha})} \sum_{i=1}^{m-2} \beta_{i} l_{i}(s), &0\leq t\leq s \leq1,\\ \frac{1}{\Gamma(\alpha+1)}(1-s)^{\alpha}-\frac{1}{\Gamma(\alpha )}(t-s)^{\alpha-1}\\ \quad{}+\frac{\alpha(\Gamma(\alpha+2)-1+(\alpha+1)t^{\alpha})}{\Gamma (\alpha+2)\sum_{i=1}^{m-2}\beta_{i}(1-\eta_{i}^{\alpha})} \sum_{i=1}^{m-2} \beta_{i} l_{i}(s), & 0\leq s \leq t\leq1. \end{cases} $$

Obviously, \(\max_{0\leq s\leq1}\sum_{i=1}^{m-2}\beta _{i}l_{i}(s)\leq1\). We denote

$$\kappa:=\min \biggl\{ 1, \frac{\sum_{i=1}^{m-2}\beta_{i}(1-\eta _{i}^{\alpha})}{\alpha}, \frac{1}{\max_{t,s\in[0,1]}G(t,s)} \biggr\} . $$

Thus, one has

$$\begin{aligned} 1-\frac{\kappa\alpha\sum_{i=1}^{m-2}\beta_{i}l_{i}(s)}{\sum_{i=1}^{m-2}\beta_{i}(1-\eta_{i}^{\alpha})}\geq0,\quad 1-\kappa G(t,s)\geq0. \end{aligned}$$
(3.1)

Theorem 3.1

Let \(f:[0,1]\times\mathbb{R}\rightarrow\mathbb{R}\) be continuous. Suppose that:

(H1):

there exist nonnegative functions \(a,b\in X\) with \(\frac {\Gamma(\alpha+1)}{2} b_{1}< 1\) such that

$$\begin{aligned} \bigl|f(t,u)\bigr|\leq a(t)+b(t)|u|, \quad\forall t\in[0,1], u\in\mathbb{R}, \end{aligned}$$

where \(b_{1}=\|b\|_{\infty}\),

(H2):

there exists a constant \(B>0\) such that

$$uf(t,u)< 0,\quad \forall t\in[0,1], |u|>B, $$
(H3):

\(f(t,u)> -\kappa u\), for all \((t,u)\in[0,1]\times[0,\infty)\),

(H4):

there exist \(r\in(0,+\infty)\), \(t_{0}\in[0,1]\), \(M\in(0,1)\) and continuous function \(h:(0,r]\rightarrow[0,\infty)\) such that \(f(t,u)\geq h(u)\) for all \(t\in[0,1]\), \(u\in(0,r]\), and \(\frac{h(u)}{u}\) is non-increasing on \((0,r]\) with

$$ \frac{h(r)}{r}\int_{0}^{1}G(t_{0},s) \,ds \geq\frac{1-M}{M}. $$

Then FBVP (1.1) has at least one solution in X.

Now, we begin with some lemmas that are useful in what follows.

Lemma 3.2

Let L be defined by (2.1), then

$$\begin{aligned}& \operatorname{Ker} L=\bigl\{ x\in X: x(t)=c, \forall t\in[0,1], c \in\mathbb {R}\bigr\} , \end{aligned}$$
(3.2)
$$\begin{aligned}& \operatorname{Im} L=\Biggl\{ y\in Y: \sum _{i=1}^{m-2}\beta_{i}\int _{0}^{1}l_{i}(s)y(s)\,ds=0\Biggr\} . \end{aligned}$$
(3.3)

Proof

By Lemma 2.5, \(D_{0^{+}}^{\alpha}x(t)=0\) has solution

$$\begin{aligned} x(t)=c_{0}+c_{1}t,\quad c_{0},c_{1}\in \mathbb{R}. \end{aligned}$$

Combining with the boundary conditions of FBVP (1.1), one sees that (3.2) holds.

For \(y\in\operatorname{Im} L\), there exists \(x\in\operatorname{dom} L\) such that \(y=Lx\in Y\). By Lemma 2.6, we have

$$\begin{aligned} x(t)=-\frac{1}{\Gamma(\alpha)}\int_{0}^{t} {(t-s)^{\alpha-1}} {y(s)}\,ds+c_{0}+c_{1}t,\quad c_{0},c_{1}\in\mathbb{R}. \end{aligned}$$

Then we get

$$\begin{aligned} x'(t)=-\frac{1}{\Gamma(\alpha-1)}\int_{0}^{t}{(t-s)^{\alpha-2}} {y(s)}\,ds+c_{1}. \end{aligned}$$

By the boundary conditions of FBVP (1.1), we see that y satisfies

$$ \int_{0}^{1}(1-s)^{\alpha-1}y(s)\,ds=\sum _{i=1}^{m-2}\beta_{i} \int _{0}^{\eta _{i}}(\eta_{i}-s)^{\alpha-1}y(s) \,ds. $$

That is,

$$\begin{aligned} \sum_{i=1}^{m-2} \beta_{i}\int_{0}^{1}l_{i}(s)y(s) \,ds=0. \end{aligned}$$
(3.4)

On the other hand, suppose \(y\in Y\) and satisfies (3.4). Let \(x(t)=-I_{0^{+}}^{\alpha}y(t)+x(0)\), then \(x\in\operatorname{dom}L\) and \(D_{0^{+}}^{\alpha}x(t)=-y(t)\). Thus, \(y\in\operatorname{Im}L\). Hence (3.3) holds. The proof is complete. □

Lemma 3.3

Let L be defined by (2.1), then L is a Fredholm operator of index zero, and the linear continuous projector operators \(P:X\rightarrow X\) and \(Q:Y\rightarrow Y\) can be defined as

$$\begin{aligned}& Px(t)=\int_{0}^{1}x(s)\,ds, \quad\forall t\in[0,1],\\& Qy(t)=\frac{\alpha}{\sum_{i=1}^{m-2}\beta_{i}(1-\eta_{i}^{\alpha })}\sum_{i=1}^{m-2} \beta_{i}\int_{0}^{1}l_{i}(s)y(s)\,ds,\quad \forall t\in[0,1]. \end{aligned}$$

Furthermore, the operator \(K_{P}:\operatorname{Im}L\rightarrow\operatorname{dom}L \cap\operatorname{Ker}P\) can be written by

$$\begin{aligned} K_{P}y(t)=\int_{0}^{1} k(t,s)y(s)\,ds,\quad \forall t\in[0,1], \end{aligned}$$

where

$$ k(t,s)= \textstyle\begin{cases} \frac{1}{\Gamma(\alpha+1)}(1-s)^{\alpha}, & 0\leq t\leq s \leq1,\\ \frac{1}{\Gamma(\alpha+1)}(1-s)^{\alpha}-\frac{1}{\Gamma(\alpha )}(t-s)^{\alpha-1}, & 0 \leq s \leq t\leq1. \end{cases} $$
(3.5)

Proof

Obviously, \(\operatorname{Im} P=\operatorname{Ker} L\) and \(P^{2}x=Px\). It follows from \(x=(x-Px)+Px\) that \(X=\operatorname{Ker} P+\operatorname {Ker} L\). By a simple calculation, one obtain \(\operatorname{Ker} L\cap\operatorname{Ker} P=\{0\}\). Thus, we get

$$\begin{aligned} X=\operatorname{Ker} L\oplus\operatorname{Ker} P. \end{aligned}$$

For \(y\in Y\), we have

$$\begin{aligned} Q^{2}y=Q(Qy)=Qy\cdot\frac{\alpha}{\sum_{i=1}^{m-2}\beta_{i}(1-\eta _{i}^{\alpha})}\sum _{i=1}^{m-2}\beta_{i}\int _{0}^{1}l_{i}(s)\,ds=Qy. \end{aligned}$$

Let \(y=(y-Qy)+Qy\), where \(y-Qy\in\operatorname{Ker} Q\), \(Qy\in\operatorname{Im} Q\). It follows from \(\operatorname{Ker} Q=\operatorname{Im} L\) and \(Q^{2}y=Qy\) that \(\operatorname{Im} Q\cap\operatorname{Im} L=\{ 0 \}\). Then one has

$$\begin{aligned} Y=\operatorname{Im} L\oplus\operatorname{Im} Q. \end{aligned}$$

Thus, we obtain

$$\begin{aligned} \operatorname{dim} \operatorname{Ker}L=\operatorname{dim} \operatorname{Im}Q =\operatorname{dim} \operatorname{Coker}L=1. \end{aligned}$$

That is, L is a Fredholm operator of index zero.

Now, we will prove that \(K_{P}\) is the inverse of \(L|_{\operatorname{dom} L\cap \operatorname{Ker} P}\). In fact, for \(y\in\operatorname{Im}L\), we have

$$\begin{aligned} K_{P}y(t)=-\frac{1}{\Gamma(\alpha)}\int_{0}^{t}(t-s)^{\alpha-1}y(s) \,ds+c_{0}, \end{aligned}$$
(3.6)

where

$$ c_{0}=\frac{1}{\Gamma(\alpha+1)}\int_{0}^{1}(1-s)^{\alpha}y(s) \,ds. $$

It is easy to see that \(LK_{P}y=y\). Moreover, for \(x\in\operatorname{dom} L\cap\operatorname{Ker} P\), we get \(x'(0)=0\) and

$$\begin{aligned} K_{P}Lx(t) =&I_{0^{+}}^{\alpha}D_{0^{+}}^{\alpha}x(t)-I_{0^{+}}^{\alpha+1} D_{0^{+}}^{\alpha}x(t)|_{t=1} \\ =&x(t)-x(0)-\int_{0}^{1}\bigl(x(s)-x(0)\bigr) \,ds \\ =&x-Px. \end{aligned}$$
(3.7)

Combining (3.6) with (3.7), we know that \(K_{P}=(L|_{\operatorname {dom}L\cap\operatorname{Ker}P})^{-1}\). The proof is complete. □

Lemma 3.4

Assume \(\Omega\subset X\) is an open bounded subset such that \(\operatorname{dom}L\cap\overline{\Omega}\neq\emptyset\), then N is L-compact on \(\overline{\Omega}\).

Proof

By the continuity of f, we see that \(QN(\overline{\Omega})\) and \(K_{P}(I-Q)N(\overline{\Omega})\) are bounded. That is, there exist constants \(A,B>0\) such that \(|(I-Q)Nx|\leq A\) and \(|K_{P}(I-Q)Nx|\leq B\), \(\forall x\in\overline{\Omega}\), \(t\in[0,1]\). Thus, one need only prove that \(K_{P}(I-Q)N(\overline{\Omega})\subset X\) is equicontinuous.

Let \(K_{P,Q}=K_{P}(I-Q)N\), for \(0\leq t_{1}< t_{2}\leq1\), \(x\in\overline {\Omega}\), we get

$$\begin{aligned} & \bigl\vert (K_{P,Q}x) (t_{2})-(K_{P,Q}x) (t_{1})\bigr\vert \\ &\quad\leq\frac{1}{\Gamma(\alpha)}\biggl\vert \int_{0}^{t_{2}}{(t_{2}-s)^{\alpha -1}} {(I-Q)Nx(s)}\,ds-\int_{0}^{t_{1}}{(t_{1}-s)^{\alpha-1}} {(I-Q)Nx(s)}\,ds \biggr\vert \\ &\quad\leq\frac{A}{\Gamma(\alpha)} \biggl[\int_{0}^{t_{1}}{(t_{2}-s)^{\alpha -1}}-{(t_{1}-s)^{\alpha-1}} \,ds+\int_{t_{1}}^{t_{2}}{(t_{2}-s)^{\alpha-1}} \,ds \biggr]\\ &\quad=\frac{A}{\Gamma( \alpha+1)}\bigl(t_{2}^{\alpha}-t_{1}^{\alpha} \bigr). \end{aligned}$$

Since \(t^{\alpha}\) is uniformly continuous on \([0,1]\), we see that \(K_{P,Q}N(\overline{\Omega})\subset X\) is equicontinuous. Thus, we see that \(K_{P,Q}N:\overline{\Omega}\rightarrow X\) is compact. The proof is completed. □

Lemma 3.5

Suppose (H1) and (H2) hold, then the set

$$\begin{aligned} \Omega_{0}=\bigl\{ x\in\operatorname{dom} L: Lx=\lambda Nx, \lambda \in(0,1)\bigr\} \end{aligned}$$

is bounded.

Proof

Take \(x\in\Omega_{0}\), then \(Nx\in\operatorname{Im} L\). By (3.2), we have

$$\begin{aligned} \sum_{i=1}^{m-2}\beta_{i}\int _{0}^{1} l_{i}(s)f\bigl(s,x(s)\bigr) \,ds=0. \end{aligned}$$

Then, by the integral mean value theorem, there exists a constant \(\xi \in(0,1)\) such that \(f(\xi,x(\xi))=0\). So, from (H2), we get \(|x(\xi )|\leq B\). By Lemma 2.6, one has

$$\begin{aligned}& x(t)= x(0) +\frac{1}{\Gamma(\alpha)}\int_{0}^{t}(t-s)^{\alpha -1}D_{0^{+}}^{\alpha}x(s)\,ds,\\& x(\xi)= x(0) +\frac{1}{\Gamma(\alpha)}\int_{0}^{\xi}( \xi-s)^{\alpha -1}D_{0^{+}}^{\alpha}x(s)\,ds. \end{aligned}$$

Thus, we get

$$\begin{aligned} x(t)-x(\xi) =&\frac{1}{\Gamma(\alpha)}\int_{0}^{t}(t-s)^{\alpha -1}D_{0^{+}}^{\alpha}x(s) \,ds\\ &{}-\frac{1}{\Gamma(\alpha)}\int_{0}^{\xi}( \xi-s)^{\alpha -1}D_{0^{+}}^{\alpha}x(s)\,ds, \end{aligned}$$

which together with (H1) implies that

$$\begin{aligned} \bigl|x(t)\bigr| \leq& \bigl|x(\xi)\bigr|+\frac{1}{\Gamma(\alpha)}\bigl\| D_{0^{+}}^{\alpha}x\bigr\| _{\infty} \cdot\frac{1}{\alpha}\bigl(t^{\beta}+\xi^{\beta}\bigr)\\ \leq&B+\frac{2}{\Gamma(\alpha+1)}\bigl\| D_{0^{+}}^{\alpha}x\bigr\| _{\infty} \\ \leq&B+\frac{2}{\Gamma(\alpha+1)}\cdot\max_{t\in[0,1]}\bigl|f\bigl(t,x(t)\bigr)\bigr|\\ \leq&B+\frac{2}{\Gamma(\alpha+1)}\bigl(\|a\|_{\infty}+b_{1}\|x \|_{\infty}\bigr), \quad \forall t\in[0,1]. \end{aligned}$$

That is,

$$\begin{aligned} \|x\|_{\infty} \leq B+\frac{2}{\Gamma(\alpha+1)}\bigl(\|a\|_{\infty}+b_{1} \|x\| _{\infty}\bigr). \end{aligned}$$

In view of \(\frac{2}{\Gamma(\alpha+1)}b_{1}<1\), there exists a constant \(D_{2}>0\) such that

$$\|x\|_{\infty} \leq D_{2}. $$

Hence, \(\Omega_{0}\) is bounded. The proof is complete. □

Proof of Theorem 3.1

Set \(C=\{x\in X: x(t)\geq 0, t\in[0,1]\}\), \(\Omega_{1}=\{x\in X: r>|x(t)|>M\|x\|_{\infty}, t\in [0,1]\}\), and \(\Omega_{2}=\{x\in X: \|x\|_{\infty}< R\}\), where \(R=\max\{ B,D_{2}\}\). Clearly, \(\Omega_{1}\), \(\Omega_{2}\) are open bounded subsets of X and

$$\overline{\Omega}_{1}=\bigl\{ x\in X: r\geq\bigl|x(t)\bigr|\geq M\|x \|_{\infty}, t\in [0,1]\bigr\} \subset\Omega_{2}. $$

From Lemma 3.3, Lemma 3.4, and Lemma 3.5, we see that the conditions (1) and (2) of Lemma 2.2 are satisfied.

Let \(\gamma x(t)=|x(t)|\) for \(x\in X\) and \(J=I\). One can see that γ is a retraction and maps subsets of \(\overline{\Omega}_{2}\) into bounded subsets of C, which means that the condition (3) of Lemma 2.2 holds.

For \(x\in\operatorname{Ker}L \cap\Omega_{2}\), we have \(x(t)\equiv c\). Let

$$H(c,\lambda)=c-\lambda|c|-\frac{\lambda\alpha}{\sum_{i=1}^{m-2}\beta _{i}(1-\eta_{i}^{\alpha})}\sum_{i=1}^{m-2} \beta_{i}\int_{0}^{1}l_{i}(s)f\bigl(s,|c|\bigr) \,ds. $$

From \(H(c,\lambda)=0\), one has \(c\geq0\). Moreover, if \(H(R,\lambda )=0\), we get

$$0\leq R(1-\lambda)=\frac{\lambda\alpha}{\sum_{i=1}^{m-2}\beta_{i}(1-\eta _{i}^{\alpha})}\sum_{i=1}^{m-2} \beta_{i}\int_{0}^{1}l_{i}(s)f(s,R) \,ds, $$

which contradicts (H2). Thus \(H(c,\lambda)\neq0\) for \(x\in\partial \Omega_{2}\), \(\lambda\in[0,1]\). Hence

$$\begin{aligned} & \operatorname{deg}\bigl(\bigl[I-(P+JQN)\gamma\bigr]|_{\operatorname{Ker}L}, \operatorname{Ker}L\cap\Omega _{2},0\bigr) \\ &\quad=\operatorname{deg}\bigl(H(c,1),\operatorname{Ker}L\cap\Omega_{2},0 \bigr) \\ &\quad=\operatorname{deg}\bigl(H(c,0),\operatorname{Ker}L\cap\Omega_{2},0 \bigr) \\ &\quad=\operatorname{deg}(I,\operatorname{Ker}L\cap\Omega_{2},0) \\ &\quad=1. \end{aligned}$$

So, the condition (4) of Lemma 2.2 holds.

Let \(x\in\overline{\Omega}_{2}\setminus\Omega_{1}\), \(t\in[0,1]\), we have

$$\begin{aligned} \Psi_{\gamma}x(t) = &\int_{0}^{1}\bigl|x(s)\bigr| \,ds+\frac{\alpha}{\sum_{i=1}^{m-2}\beta_{i}(1-\eta_{i}^{\alpha})}\sum_{i=1}^{m-2} \beta_{i}\int_{0}^{1}l_{i}(s)f \bigl(s,\bigl|x(s)\bigr|\bigr)\,ds \\ &{}+\int_{0}^{1}k(t,s) \Biggl[f\bigl(s,\bigl|x(s)\bigr| \bigr)-\frac{\alpha}{\sum_{i=1}^{m-2}\beta_{i}(1-\eta_{i}^{\alpha})}\sum_{i=1}^{m-2} \beta_{i}\int_{0}^{1}l_{i}( \tau)f\bigl(\tau,\bigl|x(\tau)\bigr|\bigr)\,d\tau \Biggr]\,ds \\ =&\int_{0}^{1}\bigl|x(s)\bigr|\,ds+\int _{0}^{1}G(t,s)f\bigl(s,\bigl|x(s)\bigr|\bigr)\,ds, \end{aligned}$$

which together with (H3) and (3.1) yields

$$\begin{aligned} \Psi_{\gamma}x(t) \geq&\int_{0}^{1}\bigl|x(s)\bigr| \,ds- \kappa\int_{0}^{1}G(t,s)\bigl|x(s)\bigr|\,ds =\int_{0}^{1}\bigl(1-\kappa G(t,s)\bigr)\bigl|x(s)\bigr| \,ds\geq0. \end{aligned}$$

Thus, the condition (7) of Lemma 2.2 holds. In addition, we can prove the condition (6) of Lemma 2.2 holds too by a similar process.

Finally, we will show that the condition (5) of Lemma 2.2 is satisfied. Let \(u_{0}(t)\equiv1\), \(t\in[0,1]\), then \(u_{0} \in C\setminus\{ 0\}\), \(C(u_{0})=\{ x\in C: x(t)>0, t\in[0,1]\}\) and we can take \(\sigma (u_{0})=1\). For \(x\in C(u_{0})\cap\partial\Omega_{1}\), we have \(x(t)>0\), \(t\in[0,1]\), \(0<\|x\|_{\infty}\leq r\), and \(x(t)\geq M\|x\|_{\infty}\), \(t\in[0,1]\). So, from (H4), we obtain

$$\begin{aligned} \Psi x(t_{0}) =&\int_{0}^{1}x(s)\,ds+ \int_{0}^{1}G(t_{0},s)f\bigl(s,x(s) \bigr)\,ds \\ \geq& M\|x\|+\int_{0}^{1}G(t_{0},s)h \bigl(x(s)\bigr)\,ds \\ =& M\|x\|+\int_{0}^{1}G(t_{0},s) \frac{h(x(s))}{x(s)}x(s)\,ds \\ \geq& M\|x\|+\frac{h(r)}{r}\int_{0}^{1}G(t_{0},s)x(s) \,ds \\ \geq& M\|x\|+\frac{h(r)}{r}\int_{0}^{1}G(t_{0},s)M \|x\|\,ds \\ \geq& M\|x\|+(1-M)\|x\| \\ =&\|x\|. \end{aligned}$$

Then the condition (5) of Lemma 2.2 holds.

Consequently, by Lemma 2.2, the equation \(Lx=Nx\) has at least one solution \(x^{*}\in C\cap(\overline{\Omega}_{2}\setminus\Omega_{1})\). Namely, FBVP (1.1) has at least one positive solution in X. The proof is complete. □

4 Example

We consider the following FBVP:

$$\begin{aligned} \left \{ \textstyle\begin{array}{@{}l} -D_{0^{+}}^{\frac{3}{2}}x(t)=\sin t-\frac{1}{10}x(t)+10+\sin x(t), \quad t\in[0,1], \\ x'(0)=0,\qquad x(1)=x(\frac{1}{2}). \end{array}\displaystyle \right . \end{aligned}$$
(4.1)

Thus, we have

$$\begin{aligned}& l(s)= \textstyle\begin{cases} \sqrt{1-s}-\sqrt{\frac{1}{2}-s}, &0\leq s \leq\frac{1}{2},\\ \sqrt{1-s}, & \frac{1}{2}\leq s\leq1, \end{cases}\displaystyle \\& G(t,s)= \textstyle\begin{cases} \frac{1}{\Gamma(\frac{5}{2})}(1-s)^{\frac{3}{2}} +\frac{3(\Gamma(\frac{7}{2})-1+(\frac{5}{2})t^{\frac {3}{2}})}{2\Gamma(\frac{7}{2})(1-(\frac{1}{2})^{\frac{3}{2}})}l(s), &0\leq t\leq s \leq1,\\ \frac{1}{\Gamma(\frac{5}{2})}(1-s)^{\frac{3}{2}}-\frac{1}{\Gamma(\frac {3}{2})}(t-s)^{\frac{1}{2}} +\frac{3(\Gamma(\frac{7}{2})-1+(\frac{5}{2})t^{\frac {3}{2}})}{2\Gamma(\frac{7}{2})(1-(\frac{1}{2})^{\frac{3}{2}})}l(s), &0\leq s\leq t \leq1. \end{cases}\displaystyle \end{aligned}$$

Moreover, \(f(t,u)\geq8-\frac{1}{10}u\geq-\frac{1}{4}u\) for all \(u\geq 0\), and \(l(s)\leq1\), \(G(t,s)\leq4\), \(\kappa=-\frac{1}{4}\). So, we can find that (H1), (H2), (H3) hold. Next, we take \(t_{0}=0\), \(h(x)=x\), and \(M=\frac{2}{3}\), thus \(G(0,s)= \frac{1}{\Gamma(\frac{5}{2})}(1-s)^{\frac{3}{2}}+\frac{3(\Gamma(\frac {7}{2})-1)}{2\Gamma(\frac{7}{2})(1-(\frac{1}{2})^{\frac{3}{2}})}l(s)\), \(0\leq s \leq1\), and \(\int_{0}^{1}G(0,s)\,ds=1\). Then (H4) is satisfied. According to the above points, by Theorem 3.1, we can conclude that FBVP (4.1) has at least one positive solution.